LSAT and Law School Admissions Forum

Get expert LSAT preparation and law school admissions advice from PowerScore Test Preparation.

User avatar
 Dave Killoran
PowerScore Staff
  • PowerScore Staff
  • Posts: 5862
  • Joined: Mar 25, 2011
|
#41326
Complete Question Explanation
(The complete setup for this game can be found here: lsat/viewtopic.php?t=8467)

The correct answer choice is (B)

If you have difficulty finishing the Logic Games section, or if you find yourself in trouble on a game, this “5 if” question format is one you should avoid. Observe the construction of the question: a Global Must Be True question stem where each of the five answer choices begins with an “if” statement. Essentially, each of the answer choices is a new scenario, and for the most part information cannot be shared among the answer choices in this question. This type of question is designed to consume time! Avoid it if you have time problems in the Logic Games section.

In a possible oversight by the test makers, this question contains an Achilles heel which allows the observant test taker to answer the question quickly. Whenever you encounter a Logic Games question where each answer choice begins with the word “if,” always make sure to check your previous work in case some of the information can be reused. In this case, the information from question #9 is duplicated in answer choice (B). Since question #9 proves that, when O lives on the second floor, L cannot live on the fourth floor, and that is what answer choice (B) states in question #11, it must be true that answer choice (B) is correct.

Honestly, it is a stroke of good fortune that the information from question #9 solves this question. Generally, on questions where each answer choice begins with “if,” using the information from previous questions would perhaps eliminate one or two answer choices at most. Of course, that would still provide a great advantage. Here, that technique answers a very time-consuming question quite quickly. Always remember to check your previous work to see if it applies to the question you are working on, especially when you know the question is specifically designed to consume time. Should you wish to complete this question, and you fail to refer to previous work, your only choice is to work through each answer until you come to one you can prove correct.
 aamcgowa
  • Posts: 10
  • Joined: Jul 17, 2021
|
#89205
Can you explain how you would work through each answer choice? I am stuck on a) as to how I would solve each one to narrow down my work if option b) was not repeated from question 9.
User avatar
 Beatrice Brown
PowerScore Staff
  • PowerScore Staff
  • Posts: 75
  • Joined: Jun 30, 2021
|
#89287
Hi AAMC! Happy to help you with this :)

For this question, you would create five separate diagrams for each answer choice. Each answer choice is a conditional statement. In conditional reasoning, when the sufficient condition occurs, the necessary condition must also occur. So for this question, if the answer choice must be true, then it is not possible to have a scenario where the sufficient condition occurs and the necessary condition does not occur.

To determine whether an answer choice must be true, we can create a diagram where the sufficient condition occurs but the necessary condition does not. If it's possible to create a diagram where the sufficient condition occurs and the necessary condition does not occur, then that answer choice does not have to be true and is incorrect. On the flipside, if it is not possible to create a diagram where the sufficient condition occurs and the necessary condition does not occur, then that answer choice does have to be true and is the correct answer.

Here's what each diagram would be for each of the five answer choices:
Answer choice (A): J is in 4 and Q is in 5
Answer choice (B): O is in 2 and L is in 4
Answer choice (C): N is in 4 and K is in 2
Answer choice (D): K is in 3 and O is in 5
Answer choice (E): P is in 4 and M is in 3.
For answer choices (A), (C), (D), and (E), it is possible to create a diagram that does not violate any of the rules. However, for answer choice (B), we cannot. If O is in 2 and L is in 4, then floors 1, 3, and 5 must have 2 variables each. But notice that we cannot then place the KP block since there are no consecutive floors open in the diagram (because L must be by itself and the second floor only has one apartment). So if O is in 2, then it must be the case that L is not in 4, because otherwise one of the rules would be violated. This means that answer choice (B) must be true and is the correct answer.

I hope this helps, and let me know if you have any other questions!

Get the most out of your LSAT Prep Plus subscription.

Analyze and track your performance with our Testing and Analytics Package.